Use Dirac function to represent constraint in an integral

constraintsdirac deltadistribution-theoryintegrationmultivariable-calculus

Hello and I want to do integral under constraint like:
$ \int_{f(x1,x2…xn) = 0}g(x1,x2,…xn)\rm dx1dx2…dxn $
some book told me I can use Dirac function to represent constraint as:
$ \int_{x1,x2…xn}g(x1,x2,…xn)\delta(f(x1,x2…xn))\rm dx1dx2…dxn $
but when I try it in practice I find something make me confused, here is an example:
I want to calculate circumference of an unit circle so I use integral as :
$ \int_{x}\int_{y}\delta(x^2 + y^2 – 1)\rm dxdy $
and we know $ \delta(f(x)) = \sum_{i=0}^N\frac{\delta(x-xi)}{\vert{\frac{\partial{f(x)}}{\partial x}} \quad\vert x=xi} $ xi is root of f(x)
so I take y as variable and $ y = \pm\sqrt{1-x^2}$
$\delta(x^2 + y^2 – 1)=\frac{\delta(y-\sqrt{1-x^2})}{2*\sqrt{1-x^2}} + \frac{\delta(y+\sqrt{1-x^2})}{2*\sqrt{1-x^2}}$
so $ \int_{x}\int_{y}\delta(x^2 + y^2 – 1)\rm dxdy = \int_{x=-1}^{1}\frac{1}{\sqrt{1-x^2}} \rm dx = \pi$
the constant 2 is missing, and if I change the constraint to $ \sqrt{x^2 + y^2} = 1 $ which is equivalent to $ x^2 + y^2 = 1$ , then
$\delta(\sqrt{x^2 + y^2} – 1)=\frac{\delta(y-\sqrt{1-x^2})}{\sqrt{1-x^2}} + \frac{\delta(y+\sqrt{1-x^2})}{\sqrt{1-x^2}}$ it's two times than before, in fact different type of constraint which have same effect can result to different scalar so it make me confused
how to integrate constraint to integral to avoid the scalar error?

Best Answer

  1. One needs to insert a gradient $${\rm Vol}(\{x \in\mathbb{R}^n|f(x)=0\})~=~ \int_{\mathbb{R}^n} \!\mathrm{d}^nx~ \color{red}{|\nabla f(x)|} \delta(f(x)) \tag{1}$$ for purely geometric reasons to ensure that the volume doesn't depend on the parametrization of the constraint function $f$.

  2. Example: The circumference of the unit-circle $\mathbb{S}^1=\{x \in\mathbb{R}^2|f(x)=0\}$. Here the constraint function is $f(x)=x_1^2+x_2^2-1$. Then $\color{red}{|\nabla f(x)|=2}$ for $x\in \mathbb{S}^1$. Hence $${\rm Vol}(\mathbb{S}^1)~\stackrel{(1)}{=}~ \int_0^{2\pi} \!\mathrm{d}\phi\int_0^{\infty} \!r\mathrm{d}r~ \color{red}{2} \delta(r^2-1)~=~2\pi. \tag{2}$$

  3. See also e.g. this related Phys.SE post.

Related Question